Những câu hỏi liên quan
Uchiha Itachi
Xem chi tiết
Lê Thị Thục Hiền
20 tháng 5 2021 lúc 13:40

\(\Leftrightarrow3x^2+2y^2+2z^2+2yz=2\)

\(\Rightarrow2\ge3x^2+2y^2+2z^2+y^2+z^2\) 

\(\Leftrightarrow2\ge3\left(x^2+y^2+z^2\right)\)

Có: \(\left(x+y+z\right)^2\le3\left(x^2+y^2+z^2\right)\le2\)

\(\Rightarrow\)\(A^2\le2\) \(\Leftrightarrow A\in\left[-\sqrt{2};\sqrt{2}\right]\)

minA=-1\(\Leftrightarrow\)\(\left\{{}\begin{matrix}x+y+z=-\sqrt{2}\\x=y=z\end{matrix}\right.\)  \(\Rightarrow x=y=z=-\dfrac{\sqrt{2}}{3}\)

maxA=1\(\Leftrightarrow\left\{{}\begin{matrix}x+y+z=\sqrt{2}\\x=y=z\end{matrix}\right.\) \(\Rightarrow x=y=z=\dfrac{\sqrt{2}}{3}\)

 

Bình luận (1)
Hoàng Khánh Chi
Xem chi tiết
Nguyễn Trần Thái Uyên
6 tháng 12 2023 lúc 23:07

Ta thấy 
72
=
2
3
.
3
2
72=2 
3
 .3 
2
  nên a, b có dạng 
{

=
2

3


=
2

.
3


a=2 
x
 3 
y
 
b=2 
z
 .3 
t
 

  với 

,

,

,


N
x,y,z,t∈N và 



{

,

}
=
3
;



{

,

}
=
2
max{x,z}=3;max{y,t}=2. 

 Theo đề bài, ta có 
2

.
3

+
2

.
3

=
42

x
 .3 
y
 +2 
z
 .3 
t
 =42

 

2


1
.
3


1
+
2


1
3


1
=
7
⇔2 
x−1
 .3 
y−1
 +2 
z−1
 3 
t−1
 =7   (*), do đó 

,

,

,


1
x,y,z,t≥1

 TH1: 



,



x≥z,y≤t. Khi đó 

=
3
,

=
2
x=3,t=2. (*) thành:

 
4.
3


1
+
3.
2


1
=
7
4.3 
y−1
 +3.2 
z−1
 =7 


=

=
1
⇔y=z=1

 Vậy 
{

=
24

=
18

a=24
b=18

  (nhận)

 TH2: KMTQ thì giả sử 



,



x≥z,y≥t. Khi đó 

=
3
,

=
2
x=3,z=2. (*) thành 

 
4.
3


1
+
2.
3


1
=
7
4.3 
y−1
 +2.3 
t−1
 =7, điều này là vô lí.

 Vậy 
(

,

)
=
(
24
,
18
)
(a,b)=(24,18) hay 
(
18
,
24
)
(18,24) là cặp số duy nhất thỏa yêu cầu bài toán.

Bình luận (0)
Nguyễn Trần Thành An
Xem chi tiết
Nguyễn  Minh Nguyêt
Xem chi tiết
Ngudheh
Xem chi tiết
Dương Thanh Ngân
Xem chi tiết
Akai Haruma
25 tháng 1 2021 lúc 10:48

Lời giải:Vì $x^2+y^2+z^2=2$ nên:

$P=\frac{x^2+y^2+z^2}{x^2+y^2}+\frac{x^2+y^2+z^2}{y^2+z^2}+\frac{x^2+y^2+z^2}{z^2+x^2}-\frac{x^3+y^3+z^3}{2xyz}$

$=3+\frac{x^2}{y^2+z^2}+\frac{y^2}{x^2+z^2}+\frac{z^2}{x^2+y^2}-\frac{x^3+y^3+z^3}{2xyz}$

$\leq 3+\frac{x^2}{2yz}+\frac{y^2}{2xz}+\frac{z^2}{2xy}-\frac{x^3+y^3+z^3}{2xyz}$

(theo BĐT AM-GM)

$=3+\frac{x^3+y^3+z^3}{2xyz}-\frac{x^3+y^3+z^3}{2xyz}=3$

Vậy $P_{\max}=3$

Dấu "=" xảy ra khi $x=y=z=\sqrt{\frac{2}{3}}$

 

Bình luận (0)
Vũ Hoài Thu
Xem chi tiết
Akai Haruma
29 tháng 5 2023 lúc 18:26

Chứng minh gì bạn?

Bình luận (0)
phạm Văn Tuấn
Xem chi tiết
Lê Song Phương
25 tháng 9 2023 lúc 5:40

Áp dụng BĐT Cô-si cho 3 số dương \(x^2,y^2,z^2\) , ta có:\(x^2+y^2+z^2\ge3\sqrt[3]{\left(xyz\right)^2}\)

\(\Leftrightarrow\left(xyz\right)^2\le\dfrac{\left(x^2+y^2+z^2\right)^3}{27}\) \(=\dfrac{1}{27}\)

\(\Leftrightarrow-\dfrac{1}{3\sqrt{3}}\le xyz\le\dfrac{1}{3\sqrt{3}}\)

 Vậy \(max_{xyz}=\dfrac{1}{3\sqrt{3}}\). Dấu "=" xảy ra khi \(x^2=y^2=z^2\) 

\(\Rightarrow\left(x,y,z\right)=\left(\dfrac{1}{\sqrt{3}},\dfrac{1}{\sqrt{3}},\dfrac{1}{\sqrt{3}}\right)\) hoặc \(\left(\dfrac{1}{\sqrt{3}},-\dfrac{1}{\sqrt{3}},-\dfrac{1}{\sqrt{3}}\right)\) và các hoán vị.

 

Bình luận (0)
Lê Cao Cường
Xem chi tiết
Bùi Đức Huy Hoàng
20 tháng 2 2022 lúc 19:48

\(\dfrac{x+y}{z}+\dfrac{y+z}{x}+\dfrac{x+z}{y}=\dfrac{x^2y+xy^2+y^2z+yz^2+x^2z+xz^2}{xyz}=\dfrac{-3xyz}{xyz}=-3\)

đề cho xy+yz+xz=0 nhân cả 2 vế với -z

=>-xyz-\(z^2\left(y+x\right)\)=0

=>-xyz=\(z^2x+z^2y\)

cmtt bạn nhân với -y và -z

=>-3xyz=\(x^2y+xy^2+y^2z+yz^2+x^2z+xz^2\)

Bình luận (0)